Tải bản đầy đủ (.pdf) (23 trang)

SH hien PTT uoc chung lon nhat boi chung nho nhat va lien he giua chung 2016 08 15

Bạn đang xem bản rút gọn của tài liệu. Xem và tải ngay bản đầy đủ của tài liệu tại đây (155.52 KB, 23 trang )

ƯỚC CHUNG LỚN NHẤT, BỘI CHUNG NHỎ NHẤT VÀ
LIÊN HỆ GIỮA CHÚNG
PHẠM THỊ THU HIỀN
Chuyên Hùng Vương - Phú Thọ
Email:
Các khái niệm ước số chung lớn nhất (GCD) và bội số chung nhỏ
nhất (LCM) là những khái niệm cơ bản của số học. Cách đây khoảng
2000 năm, con người đã biết đến thuật toán Euclid nổi tiếng và có lẽ là
thuật toán cổ nhất của toán học để tìm ước số chung lớn nhất của hai
số nguyên a và b với b = 0. Với mục đích tìm hiểu sâu hơn sự vận dụng
của hai khái niệm này, báo cáo tập trung trình bày một số tính chất của
ước số chung lớn nhất, bội số chung nhỏ nhất và mối liên hệ giữa chúng
cùng các ví dụ điển hình được chọn lọc từ các cuộc thi chọn học sinh
giỏi của các nước.

1

Ước chung lớn nhất

Định nghĩa 1.1. Cho hai số nguyên a và b trong đó ít nhất một số khác
0 . Số nguyên dương d được gọi là ước số chung lớn nhất của a và b nếu:
(i) d | a và d | b (d là ước số chung của a và b),
(ii) Nếu c | a và c | b thì c | d.
Kí hiệu ước số chung lớn nhất của a và b là gcd (a, b) hoặc (a, b). Nếu
gcd (a, b) = 1 thì ta nói a và b là hai số nguyên tố cùng nhau.
Tính chất 1.2. Sau đây là một số tính chất của ước số chung lớn nhất
(i) gcd (a, b) = d ⇔ gcd

a b
;
d d



= 1.

(ii) Cho c là một số nguyên dương. Khi đó ta có gcd (ca, cb) = c. gcd (a, b) .
(iii) gcd (a, b) = 1 và b | ac thì b | c.
1


(iv) Nếu gcd (a, b) = 1 và gcd (a, c) = 1 thì gcd (a, bc) = 1.
(v) Nếu a = bq + r thì gcd (a, b) = gcd (b, r).
(vi) Nếu a = pα1 1 .pα2 2 ...pαk k và b = pβ1 1 .pβ2 2 ...pβk k với αi , βi ∈ N, αi + βi ≥ 1,
min{α1 ,β1 } min{α2 ,β2 }
min{αk ,βk }
i = 1, 2, ..., k, thì gcd (a, b) = p1
.p2
.
...pk
Khái niệm ước số chung lớn nhất được mở rộng tự nhiên cho trường
hợp có hơn hai số theo cách như sau
gcd (a1 , a2 , ..., an−1, an ) = gcd (gcd (a1 , a2, ..., an−1) , an ) .
Định lí 1.3 (Bachet - Bezout). Với hai số nguyên dương a và b, luôn
tồn tại các số nguyên x và y sao cho ax + by = gcd (a, b).
Định lí 1.4. Với các số nguyên a, b ta có a2 , b2 = (a, b)2 .
Chứng minh. Trước hết ta chứng minh bổ đề: Cho các số nguyên a, b, c
khác 0. Khi đó ta có
(a, bc) = (a, (a, b) c) .
Ta có (a, (a, b) c) | ((a, b) c) | bc. Do đó (a, (a, b) c) là ước của a và bc, suy
ra (a, (a, b) c) | (a, bc).
Mặt khác, (a, bc) là ước của a và bc nên nó là ước của ac và bc. Vì
vậy (a, bc) | (ac, bc) = c (a, b). Ta có (a, bc) là ước của a và c (a, b) nên

(a, bc) | (a, (a, b) c).
Vậy (a, bc) = (a, (a, b) c) .
Giả sử m, n là hai số nguyên tố cùng nhau, tức là (m, n) = 1. Áp dụng
bổ đề vừa chứng minh ta có
m2 , n2 = m2 , m2 , n n = m2 , (n, (m, n) m) n .
Vì (m, n) = 1 nên m2 , (n, (m, n) m) n = m2 , n . Lại áp dụng bổ đề ta

m2 , n = (n, (m, n) m) = 1.
Vì vậy (m, n) = 1 suy ra m2 , n2 = 1.
Ta có theo tính chất 1.2. (i)
a
b
,
(a, b) (a, b)
2

=1


suy ra

b2
a2
,
(a, b)2 (a, b)2

= 1.

Từ đó theo tính chất 1.2. (ii) ta có a2 , b2 = (a, b)2 .
Ví dụ 1.5. Cho hai số nguyên a, b nguyên tố cùng nhau. Chứng minh

rằng
a + b, a2 − ab + b2 = 1 hoặc 3.
Lời giải. Giả sử d = a + b, a2 − ab + b2 . Ta có
d | (a + b)2 − a2 + ab − b2 = 3ab.
Do đó d | 3b (a + b) − 3ab = 3b2. Tương tự d | 3a2. Do đó
d | 3a2 , 3b2 = 3 a2 , b2 = 3 (a, b)2 = 3.
Từ đó ta có điều phải chứng minh.
Ví dụ 1.6 (1997 Russian Mathematical Olympiad). Tìm tất cả các bộ
ba số tự nhiên m, n, l thỏa mãn đồng thời các điều kiện sau
m + n = (m, n)2 , m + l = (m, l)2 , n + l = (n, l)2 .
Lời giải. Đặt d = gcd (m, n, l). Khi đó ta có m = dm1 , n = dn1, l = dl1
với gcd (m1 , n1, l1) = 1.
Đặt dmn = gcd (m1 , n1). Theo giả thiết ta có d (m1 + n1) = d2d2mn . Suy
ra m1 + n1 = dd2mn . Tương tự như vậy với dnl và dlm ta có
2 (m1 + n1 + l1) = d d2mn + d2nl + d2lm .
Ta có

d
là ước của m1 + n1 + l1 và cũng là ước của m1 + n1 nên
gcd (d, 2)

d
là ước của l1 . Tương tự ta có nó cũng là ước của m1 và n1. Vì
gcd (d, 2)
d
= 1, và do đó d ≤ 2.
gcd (m1 , n1, l1) = 1 nên
gcd (d, 2)
Mặt khác dmn , dnl , dlm đôi một nguyên tố cùng nhau, suy ra
l1 = l2dlmdln , m1 = m2 dlmdmn , n1 = n2 dnmdnl

3


và ta có
m2 dlm + n2 dln = ddmn .
Không làm mất tổng quát ta giả sử dmn = min {dmn , dlm, dln} ta có
2dmn ≥ ddmn = m2 dlm + n2dln ≥ dlm + dln ≥ 2dmn.
Từ bất đẳng thức trên suy ra d = 2, m2 = n2 = 1 và dmn = dlm = dln
mà các số này nguyên tố cùng nhau nên chúng bằng 1. Từ đó suy ra
m1 = n1 = 1 và từ l1 + m1 = dd2lm có l1 = 1.
Vậy m = n = l = 2.
Ví dụ 1.7. Cho dãy số Fibonacci F1 = F2 = 1, Fn+2 = Fn+1 + Fn, với
n ≥ 1. Chứng minh rằng
(i) gcd (Fn+1, Fn) = 1,
(ii) Nếu s, t là các số nguyên s ≥ 1, t ≥ 0 thì Fs+t = Fs−1Ft + Fs Ft+1,
(iii) Nếu n | m thì Fn | Fm ,
(iv) gcd (Fm , Fn) = Fgcd(m,n) .
Lời giải. (i) Đặt d = (Fn , Fn+1). Ta có d | Fn và d | Fn+1 nên d |
Fn+1 − Fn = Fn−1. Do đó d | Fn − Fn−1 = Fn−2. Tiếp tục quá trình như
vậy ta suy ra d | F1 = 1. Vì vậy ta có d = 1.
(ii) Ta cố định t và chứng minh quy nạp theo s.
Với s = 1 ta có Ft+1 = F0Ft + F1 Ft+1 là hiển nhiên.
Giả sử s > 1 và Fs−k+t = Fs−k−1Ft + Fs−k Ft+1 với mọi k thỏa mãn
1 ≤ k ≤ s − 1.
Khi đó ta có
Fs+t = Fs+t−1 + Fs+t−2
= Fs−1+t + Fs−2+t
= Fs−2Ft + Fs−1Ft+1 + Fs−3Ft + Fs−2Ft+1
= Ft (Fs−2 + Fs−3) + Ft+1 (Fs−1 + Fs−2)
= Ft Fs−1 + Ft+1Fs

4


Đến đây ta kết thúc chứng minh.
(iii) Cố định n. Ta chứng minh quy nạp theo k.
Nếu k = 1 thì rõ ràng Fn | Fn .
Giả sử Fn | Fkn. Ta đi chứng minh Fn | F(k+1)n . Thật vậy, đặt s = n,
t = kn vào đẳng thức (ii) ta có
F(k+1)n = Fn+kn = Fn−1Fkn + Fn Fkn+1.
Từ đó suy ra Fn | F(k+1)n .
(iv) Đặt d = (Fn , Fm) và a = (m, n). Ta đi chứng minh d | Fa và Fa | d.
Thật vậy, vì a | m và a | n nên Fa | Fm , Fa | Fn suy ra Fa | d.
Nếu d = 1 thì hiển nhiên d | Fa . Trong trường hợp d > 1. Theo định
lí Bezout, tồn tại hai số nguyên x, y sao cho xm + yn = a. Hiển nhiên
không thể xảy ra trường hợp cả x và y là nguyên âm. Vì a | m, a | n nên
a ≤ m, a ≤ n. x, y cũng không thể cùng là các số nguyên dương vì nếu
như vậy thì a = xm + yn ≥ m + n, mâu thuẫn. Do đó, x và y trái dấu
nhau, không mất tổng quát ta giả sử x ≤ 0, y > 0.
Ta có Fyn = Fa−xm = Fa−1F−xm + FaF−xm+1.
Ta có n | ny, m | (−xm), do đó Fn | Fyn , Fm | F−xm . Từ đó suy ra d | Fyn
và d | F−xm . Vì thế d | FaF−xm+1.
Dễ dàng thấy được d | Fm | F−xm . Nếu (d, F−xm+1) = k > 1 thì k |
F−xm+1 và k | F−xm . Mà hai số Fibonacci liên tiếp nguyên tố cùng nhau.
Ta có mâu thuẫn.
Từ đó (d, F−xm+1) = 1, suy ra d | Fa . Đến đây ta hoàn thành chứng
minh.
Ví dụ 1.8. Chứng minh rằng không tồn tại số Fibonacci lẻ nào chia hết
cho 17.
Lời giải. Đặt d = (17, Fn), hiển nhiên d là số nguyên dương lẻ. Ta có
(17, Fn) = (34, Fn) = (F9, Fn) = F(9,n) = F1, F3 hoặc F9. Điều đó có

nghĩa d = 1, 2 hoặc 34. Vì d lẻ nên ta có d = 1. Từ đó ta có điều phải
chứng minh.
Ví dụ 1.9. Xét các số Fermat fn = 22 + 1. Chứng minh rằng với các
số nguyên dương m, n, m > n ta có (fm, fn ) = 1.
n

5


Lời giải. Ta có với mọi số nguyên dương m
m

fm − 2 = 2 2 − 1
= 22

n−1

.2

−1

= (fm−1 − 1)2 − 1

2
= fm−1
− 2fm−1

= fm−1 (fm−1 − 2)

= fm−1fm−2 (fm−2 − 2)


= ···

= fm−1fm−2 · · · f1f0 .
Giả sử d = (fm , fn) thì d | fn | fm − 2. Do đó d | [fm − (fm − 2)] = 2.
Rõ ràng d là số nguyên dương lẻ. Vì vậy fm và fn nguyên tố cùng nhau.
Ví dụ 1.10. Cho f (x) = x3 −x+1. Kí hiệu an (x) = f (f (... (f (x)) ...)),
n lần f . Chứng minh rằng các số trong dãy m, a1 (m) , a2 (m) , ... đôi một
nguyên tố cùng nhau với mọi số tự nhiên m > 1.
Lời giải. Ta có f (x) − 1 = x3 − x = (x − 1) x (x + 1). Đặt ai = ai (m).
Khi đó ta có
an+k − 1 = (an+k−1 − 1) an+k−1 (an+k−1 + 1)

= (an+k−2 − 1) an+k−2 (an+k−2 + 1) an+k−1 (an+k−1 + 1)

= ···

= (an − 1) an an+1 · · · an−k+1 (an + 1) (an+1 + 1) · · · (an+k−1 + 1) .
Do đó với các số tự nhiên i, j và i > j > 1 ta có aj | (ai − 1). Giả sử
d = (ai , aj ) thì d | [ai − (ai − 1)] = 1. Do đó d = 1. Vậy các số trong dãy
m, a1 (m) , a2 (m) , ... đôi một nguyên tố cùng nhau với mọi số tự nhiên
m > 1.
Ví dụ 1.11. Cho f (x) là một đa thức với các hệ số nguyên. Kí hiệu
a0 = 0, an = f (an−1 ) với mọi n ≥ 1. Chứng minh rằng (am , an ) = |a(m,n) |.
Khi f (x) =

9

xi + 2, hãy tìm (a2016, a2014).


i=2

6


Lời giải. Đẳng thức
(1.1)

(am , an ) = |a(m,n) |

hiển nhiên đúng khi m = 0 hoặc n = 0. Bây giờ ta xét m, n > 0.
Để chứng minh đẳng thức (1.1) trước hết ta chứng minh tính chất
.
anq ..an

(1.2)

đúng với mọi n, q ∈ N∗ .
Chứng minh tính chất (1.2) bằng quy nạp theo q.
Hiển nhiên (1.2) đúng khi q = 1.
.
Giả sử (1.2) đúng với q − 1 tức là ta có an(q−1) ..an . Ta cần chứng minh
.
anq ..an .
Với mỗi số nguyên dương k ta đặt fk (x) = f (f (... (f (x)) ...)), k lần f .
Ta có
.
anq − an = fn an(q−1) − fn (0) ..an(q−1) .
Kết hợp với giả thiết quy nạp, ta có (1.2) đúng với q.
Bây giờ để chứng minh (1.1), tương tự như thuật toán Euclid, ta chỉ ra

rằng, nếu m = nq + r thì

(1.3)

(am , an ) = (an , ar )
Thật vậy ta có

.
am − ar = fr (anq ) − fr (0) ..anq .

Kết hợp với tính chất (1.2) ta có

.
am − ar ..an
Từ đó ta có đẳng thức (1.3) đúng.
Khi f (x) =

9

xi + 2 ta có

i=2

(a2016, a2014) = a2 = 1024.
Ví dụ 1.12. Cho n là một số tự nhiên. Tìm ước chung lớn nhất của
2n
1

,


2n
3

, ...,
7

2n
2n − 1

.


Lời giải. Ta có
n

k=1

2n
2k − 1

= 22n−1,

suy ra ước chung lớn nhất cần tìm phải có dạng 2a . Ta giả sử n = 2l m
với m là số tự nhiên lẻ. Mặt khác, ước chung lớn nhất của các số trên
2n
là ước của
= 2n nên nó nhỏ hơn hoặc bằng 2l+1. Ta đi chứng
1
l+1
minh 2 là ước số chung lớn nhất cần tìm. Ta có

2l+1m
2k − 1
Nhưng 2k − 1 ∤ 2

l+1

2l+1m 2l+1m − 1
=
2k − 2
2k − 1

với k > 1 nên 2

l+1

lớn nhất của các số đã cho bằng 2l+1.

2

|

2l+1m
2k − 1

.
. Từ đó ước chung

Bội chung nhỏ nhất

Định nghĩa 2.1. Cho hai số nguyên a và b khác 0. Số nguyên dương k

được gọi là bội số chung nhỏ nhất của a và b nếu:
(i) a | k, b | k (k là bội số chung của a và b),
(ii) Nếu a | t và b | t thì k | t.
Kí hiệu bội số chung nhỏ nhất của a và b là lcm (a, b) hoặc [a, b].
Tính chất 2.2. Sau đây là một số tính chất của bội số chung nhỏ nhất
(i) Nếu lcm (a, b) = k thì gcd

k k
,
a b

= 1.

(ii) t.lcm (a, b) = lcm (ta, tb).
(iii) Nếu d | a, d | b và d > 0 thì lcm

a b
,
d d

=

lcm (a, b)
.
d

(iv) Nếu a = pα1 1 .pα2 2 ...pαk k và b = pβ1 1 .pβ2 2 ...pβk k với αi , βi ∈ N, αi + βi ≥ 1,
max{α1 ,β1 } max{α2 ,β2 }
max{αk ,βk }
i = 1, 2, ..., k, thì lcm (a, b) = p1

.p2
...pk
.
8


Khái niệm bội số chung nhỏ nhất được mở rộng tự nhiên cho trường
hợp có hơn hai số theo cách như sau
lcm (a1 , a2 , ..., an−1, an ) = lcm (lcm (a1 , a2, ..., an−1) , an ) .
Ví dụ 2.3. Chứng minh rằng
lcm (1, 2, ..., 200) = lcm (101, 102, ..., 200) .
Lời giải. Ta đi chứng minh bài toán tổng quát: Với mọi số nguyên dương
n ta có
lcm (1, 2, ..., 2n) = lcm (n + 1, n + 2, ..., 2n) .
Đặt A = lcm (1, 2, ..., 2n), B = lcm (n + 1, n + 2, ..., 2n). Hiển nhiên ta
.
có A..B.
.
.
Bây giờ ta sẽ chứng minh B ..A. Ta đi chứng minh B ..k với mọi k =
1, 2, ..., 2n.
.
Nếu k ∈ {n + 1, n + 2, ..., 2n} thì hiển nhiên B ..k.
Nếu k ∈ {1, 2, ..., n} thì trong k số nguyên dương liên tiếp n + 1, n +
2, ..., n + k có một số chia hết cho k. Giả sử đó là a. Ta có
n + 1 ≤ a ≤ n + k ≤ 2n,

.
do đó B ..k.
.

.
Tóm lại, với mọi k ∈ {1, 2, ..., 2n} ta có B ..k. Vì vậy B ..A.
Vậy A = B.
Ví dụ 2.4. Hỏi có bao nhiêu bộ ba số nguyên dương (a, b, c) thỏa mãn
lcm (a, b) = 1000, lcm (b, c) = 2000, lcm (c, a) = 2000.
Lời giải. Ta thấy cả hai số 1000 và 2000 đều có dạng 2m 5n nên các số
a, b, c đều phải có dạng này. Đặt a = 2m1 5n1 , b = 2m2 5n2 , c = 2m3 5n3 ,
trong đó mi , ni là các số nguyên không âm với i = 1, 2, 3.
Từ giả thiết ta có
max {m1 , m2} = 3, max {m2 , m3} = 4, max {m3 , m1 } = 4

(2.1)

max {n1, n2} = 3, max {n2 , n3} = 3, max {n3 , n1} = 3.

(2.2)



9


Từ (2.1) ta có m3 = 4, hai số m1 , m2 có một số bằng 3 và một số nhận
một trong các giá trị 0, 1, 2, 3. Có 7 bộ (m1 , m2 , m3) thỏa mãn đó là
(0, 3, 4), (1, 3, 4), (2, 3, 4), (3, 0, 4), (3, 1, 4), (3, 2, 4) và (3, 3, 4).
Từ (2.2) ta có hai trong ba số n1, n2, n3 bằng 3 và số còn lại nhận một
trong các giá trị 0, 1, 2, 3. Ta có 10 bộ (n1, n2, n3) thỏa mãn đó là (3, 3, 0),
(3, 3, 1), (3, 3, 2), (3, 0, 3), (3, 1, 3), (3, 2, 3), (0, 3, 3), (1, 3, 3), (2, 3, 3) và
(3, 3, 3).
Như vậy ta có tất cả 7.10 = 70 bộ số thỏa mãn đề bài.

Ví dụ 2.5 (Putnam 1980). Hỏi có bao nhiêu bộ 4 số (a, b, c, d) thỏa mãn
3r 7s = [a, b, c] = [b, c, d] = [c, d, a] = [d, a, b] .
Lời giải. Vì sự phân tích thành các nhân tử nguyên tố của một số nguyên
là duy nhất nên mỗi số a, b, c, d đều phải có dạng 3mi 7ni với 0 ≤ mi ≤ r,
0 ≤ ni ≤ s, i = 1, 2, 3, 4.
Hơn nữa, mi bằng r đối với ít nhất hai số trong 4 số mi . Tương tự ni
4
bằng s đối với ít nhất hai trong 4 số ni . Ta có
r2 = 6r2 cách chọn
2
hai trong bốn số mi bằng r và hai số mũ mi còn lại nhận các giá trị
4
trong tập {0, 1, ..., r − 1}, có
r = 4r cách chọn ba trong bốn số
3
mi bằng r và một số mi còn lại nhận giá trị trong tập {0, 1, ..., r − 1}, có
4
= 1 cách chọn cả bốn số mi bằng r. Như vậy ta có 1 + 4r + 6r2
4
cách chọn ít nhất hai trong bốn số a, b, c, d có lũy thừa của 3 bằng r.
Tương tự như vậy ta có 1 + 4s + 6s2 cách chọn ít nhất hai trong bốn số
a, b, c, d có lũy thừa của 7 bằng s. Như vậy ta có
1 + 4s + 6s2

1 + 4r + 6r2
bộ bốn số thỏa mãn đề bài.

Ví dụ 2.6 (AMM E2686). Cho số nguyên dương n. Chứng minh rằng
(n + 1) lcm


n
0

,

n
1

, ...,

n
n
10

= lcm (1, 2, ..., n + 1) . (2.3)


Lời giải. Trước hết ta nhắc lại định lí: Giả sử n là số nguyên dương.
Khi đó lũy thừa của số nguyên tố p xuất hiện trong phân tích ra thừa
số nguyên tố của n! là
n
n
n
+ 2 + 3 + ···
p
p
p
Giả sử p là một số nguyên tố, p ≤ n + 1 và α (tương ứng β) là số mũ
lớn nhất của p ở vế trái (tương ứng vế phải) của (2.3). Chọn số r thỏa
mãn pr ≤ n + 1 < pr+1 . Rõ ràng β = r.

m
Ta cần chứng minh rằng, nếu pr ≤ m < pr+1 thì pr+1 ∤
với
k
0 ≤ k ≤ m (*).
Thật vậy, lũy thừa của p xuất hiện trong sự phân tích ra thừa số nguyên
m
tố của

k
r

γ=
s=1

m
k
m−k


ps
ps
ps

.

Mỗi hạng tử trong tổng trên bằng 0 hoặc 1, ta có γ ≤ r, do đó (*) đúng.
Với 0 ≤ k ≤ n, đặt
ak = (n + 1)


n
k

= (n − k + 1)

n+1
k

n+1
k+1

= (k + 1)

Từ (*) ta có pr+1 không là ước của các số nguyên

n
k

,

n+1
k

.
,

n+1
. Vì vậy pr+1 là ước của ak khi và chỉ khi p là ước của cả ba
k+1
số n + 1, n − k + 1, k + 1. Từ đó suy ra

p | [(n + 1) − (n − k + 1) − (k + 1)] = −1,
mâu thuẫn. Vì vậy pr+1 ∤ ak .
Mặt khác với k = pr − 1 ta có k ≤ n nên ak = (k + 1)
hết cho pr .
Vậy β = r = α. Từ đó suy ra điều phải chứng minh.
11

n+1
k+1

chia


Ví dụ 2.7 (APMO 1998). Tìm số nguyên n lớn nhất thỏa mãn n chia

hết cho tất cả các số nguyên nhỏ hơn 3 n.

Lời giải. Ta thấy 7 < 3 420 < 8 và 420 = lcm {1, 2, 3, 4, 5, 6, 7}. Ta đi
chứng minh 420 là số cần tìm.
Giả sử n > 420 là số nguyên thỏa mãn chia hết cho tất cả các số nguyên


nhỏ hơn 3 n. Ta có 3 n > 7 nên n chia hết cho 420 = lcm {1, 2, 3, 4, 5, 6, 7},

do đó n ≥ 840 và 3 n > 9. Do đó n chia hết 2520 = lcm {1, 2, 3, 4, 5, 6, 7, 8, 9}

và ta lại có 3 n > 13. Giả sử m là số nguyên dương lớn nhất nhỏ hơn


3

n. Khi đó ta có m < 3 n ≤ m + 1. Ta có m ≥ 13 và n chia hết cho
lcm {1, 2, 3, 4, 5, 6, 7}. Mặt khác, ta lại có
lcm (m − 3, m − 2, m − 1, m) ≥

m (m − 1) (m − 2) (m − 3)
6

vì chỉ có 2 và 3 là ước chung của bốn số trên.
Từ đó ta có
m (m − 1) (m − 2) (m − 3)
≤ n ≤ (m + 1)3 .
6
Suy ra
m≤6 1+

2
m−1

1+

3
m−2

1+

4
.
m−3

Vế trái của bất đẳng thức trên là hàm đơn điệu tăng của biến m, còn vế

phải là hàm đơn điệu giảm của biến m. Với m = 13 ta có
13.12.11.10 = 17160 > 6.143
do vậy bất đẳng thức trên không đúng với mọi m ≥ 13.
Vậy không có số m > 420 nào thỏa mãn điều kiện đề bài.
Ví dụ 2.8 (1951 Russian Mathematics Olympiad). Cho các số nguyên
dương a1 , a2 , ..., an nhỏ hơn 1000 và lcm (ai , aj ) > 1000 với mọi i, j, i = j.
Chứng minh rằng
n
1
< 2.
a
i
i=1
12


1000
1000
thì có m bội số của a là a, 2a, ..., ma
m+1
m
1000
, 1000 ,
không vượt quá 1000. Giả sử k1 là số các ai nằm trong khoảng
2
1000 1000
k2 là số các ai nằm trong khoảng
,
,...Như vậy ta có k1 +

3
2
2k2 + 3k3 + · · · các số nguyên, không lớn hơn 1000, là bội của ít nhất
một số ai nào đó. Vì các bội này là phân biệt (vì lcm (ai , aj ) > 1000)
nên ta có
Lời giải. Nếu

k1 + 2k2 + 3k3 + · · · < 1000
⇒ 2k1 + 3k2 + 4k3 + · · · < (k1 + 2k2 + 3k3 + · · ·) + (k1 + k2 + k3 + · · ·)
< 1000 + n < 2000.
Do đó,
n

i=1

1
2
3
4
2k1 + 3k2 + 4k3 + · · ·
≤ k1
+ k2
+ k3
+ ··· =
< 2.
ai
1000
1000
1000
1000


Ví dụ 2.9. Cho 0 < a1 < a2 < ... < an ≤ 2n là các số nguyên thỏa mãn
bội số chung nhỏ nhất của hai số bất kì trong chúng đều lớn hơn 2n.
Chứng minh rằng
2n
a1 >
.
3
( ⌊α⌋ là kí hiệu phần nguyên của số thực α)
Lời giải. Rõ ràng, trong các số trên không tồn tại cặp số nào mà số
này chia hết cho số kia (vì nếu trái lại thì bội chung nhỏ nhất của chúng
nhỏ hơn hoặc bằng 2n). Ta viết ak = 2tk Ak với Ak là số lẻ. Ta thấy các
giá trị Ak là phân biệt. Mặt khác từ 1 đến 2n ta có n số lẻ phân biệt.
Do đó các giá trị Ak là các số lẻ từ 1 đến 2n theo một thứ tự nào đó.
Ta xét a1 = 2t1 A1.
2n
Nếu a1 ≤
thì 3a1 = 2t1 3A1 ≤ 2n tức là 3A1 < 2n. Do đó 3A1 là
3
một số lẻ nhỏ hơn 2n, tức là 3A1 = Aj nào đó. Như vậy aj = 2tj 3A1.
Khi đó [a1 , aj ] = 2t1 3A1 = 3a1 ≤ 2n hoặc [a1 , aj ] = 2tj 3A1 = aj ≤ 2n,
2n
.
mâu thuẫn với giả thiết. Vậy điều giả sử là sai, tức là ta có a1 >
3
13


Ví dụ 2.10. Cho a0 < a1 < a2 < · · · < an là các số nguyên dương.
Chứng minh rằng

1
1
1
1
+
+ ··· +
≤ 1 − n.
lcm (a0 , a1 ) lcm (a1 , a2 )
lcm (an−1, an )
2

(2.4)

Lời giải. Ta chứng minh quy nạp theo n.

1
1
≤ 1− .
lcm (a0 , a1)
2
Giả sử (2.4) đúng với n = k, tức là nếu a0 < a1 < a2 < · · · < ak thì ta

1
1
1
1
+
+ ··· +
≤ 1 − k.
lcm (a0 , a1) lcm (a1 , a2)

lcm (ak−1, ak )
2

Với n = 1, ta có lcm (a0, a1) ≥ lcm (1, 2) = 2. Do đó

Ta đi chứng minh (2.4) cũng đúng với n = k + 1. Giả sử a0 < a1 < a2 <
· · · < ak < ak+1 là các số nguyên dương. Ta xét hai trường hợp.
Trường hợp 1. ak+1 ≥ 2k+1. Ta có lcm (ak , ak+1) ≥ ak+1 ≥ 2k+1. Theo giả
thiết quy nạp ta có
1
1
1
1
+
+ ··· +
+
lcm (a0 , a1 ) lcm (a1 , a2)
lcm (ak−1 , ak ) lcm (ak , ak+1)
1
1
1
≤ 1 − k + k+1 = 1 − k+1 ,
2
2
2
Trường hợp 2. ak+1 < 2k+1. Khi đó ta có
1
gcd (ai−1 , ai)
ai − ai−1
1

1
=

=
− .
lcm (ai−1 , ai)
ai−1ai
ai−1ai
ai−1 ai
Cộng vế các bất đẳng thức như trên với i từ 1 đến k + 1 ta được
1
1
1
1
+
+ ··· +
+
lcm (a0 , a1 ) lcm (a1 , a2)
lcm (ak−1 , ak ) lcm (ak , ak+1)
1
1
1

≤ 1 − k+1 .

a0 ak+1
2
Theo phương pháp quy nạp ta được điều phải chứng minh.
Ví dụ 2.11. Cho các số nguyên dương không vượt quá số nguyên dương
m không đổi cho trước. Chứng minh rằng nếu mọi số nguyên dương nhỏ

hơn hoặc bằng m không chia hết cho cặp hai số nào trong các số được
3
cho ở trên, thì tổng nghịch đảo của các số đã cho nhỏ hơn .
2
14


Lời giải. Theo giả thiết mọi số nguyên dương nhỏ hơn hoặc bằng m
không chia hết cho cặp hai số nào trong các số được cho ở trên nên bội
chung nhỏ nhất của hai số bất kì trong các số được cho lớn hơn m. Giả
m
bội của xi trong
sử cho n số, kí hiệu x1, x2, ..., xn. Với mỗi i, ta có
xi
các số 1, 2, ..., m, ( ⌊α⌋ là kí hiệu phần nguyên của số thực α). Không có
số nào trong chúng là bội của xj với j = i vì bội chung nhỏ nhất của xi
và xj lớn hơn m. Như vậy ta có
m
m
m
+
+ ··· +
x1
x2
xn
phần tử đôi một khác nhau của tập hợp {1, 2, ..., m} chia hết cho một
trong các số x1, x2, ..., xn. Trong các phần tử này không có phần tử nào
bằng 1 (trừ trường hợp n = 1, khi đó điều phải chứng minh là hiển
nhiên). Do đó
m

m
m
+
+ ··· +
≤ m − 1.
x1
x2
xn
Với mọi i ta lại có
m

m
m
<
+ 1, suy ra
xi
xi
1
1
1
+
+ ··· +
x1 x2
xn

Ta còn phải chứng minh n ≤

< m + n − 1.

m+1

, khi đó ta suy ra được
2

1
1
1
n−1 3
+
+ ... +
<1+
< .
x1 x2
xn
m
2
Thật vậy, ta có các ước số lẻ lớn nhất của các số x1, x2, ..., xn là phân
biệt vì nếu trái lại có hai số có cùng ước số lẻ lớn nhất thì một số trong
chúng là bội của số còn lại, trái với giả thiết. Suy ra n không vượt quá
m+1
số các số lẻ trong các số 1, 2, ..., m. Do đó n ≤
.
2

15


3

Liên hệ giữa ước chung lớn nhất và bội chung nhỏ
nhất


Định lí 3.1. Cho m, n là hai số nguyên dương. Khi đó ta có
mn = gcd (m, n) .lcm (m, n) .
Chứng minh. Giả sử m = pα1 1 .pα2 2 ...pαk k và n = pβ1 1 .pβ2 2 ...pβk k với αi , βi ∈
N, αi + βi ≥ 1, i = 1, 2, ..., k. Khi đó ta có
min{α1 ,β1 }

gcd (m, n) = p1

max{α1 ,β1 }

lcm (m, n) = p1

min{α2 ,β2 }

.p2

max{α2 ,β2 }

.p2

min{αk ,βk }

...pk

.

max{αk ,βk }

...pk


.

Ta có
min{α1 ,β1 }+max{α1 ,β1 }

gcd (m, n) .lcm (m, n) = p1

min{αk ,βk }+max{αk ,βk }

...pk

= pα1 1 +β1 ...pαk k +βk
= mn.

Nhận xét 3.2. Cho các số nguyên dương a1 , a2 , ..., an, n ≥ 3 thì đẳng
thức
a1 a2 ...an = gcd (a1 , a2 , ..., an) .lcm (a1 , a2 , ..., an)
nói chung là không đúng. Chẳng hạn
gcd (3, 3, 3) .lcm (3, 3, 3) = 3.3 = 9 < 3.3.3 = 27.
Ví dụ 3.3. Cho m, n là các số nguyên dương thỏa mãn
lcm (m, n) + gcd (m, n) = m + n.
Chứng minh rằng một trong hai số đó chia hết cho số còn lại.
Lời giải. Cách 1. Đặt d = gcd (m, n). Khi đó ta có m = ad, n = bd với
gcd (a, b) = 1 và
lcm (m, n) =

mn
= abd.
gcd (m, n)

16


Theo bài ra ta có phương trình abd + d = ad + bd hay ab − a − b + 1 = 0.
Từ đó ta có (a − 1) (b − 1) = 0, suy ra a = 1 hoặc b = 1. Như vậy ta có
m = d, n = bd = bm hoặc n = d, m = an.
Cách 2. Ta có lcm (m, n) . gcd (m, n) = m.n, suy ra lcm (m, n) và gcd (m, n)
là các nghiệm của phương trình
x2 − (m + n) x + mn = 0.

Dễ thấy hai nghiệm của phương trình là m và n. Do đó
{lcm (m, n) , gcd (m, n)} = {m, n} .

Từ đó ta có điều phải chứng minh.

Ví dụ 3.4 (St. Petersburg 2001). Cho m, n là các số nguyên dương,
m > n. Chứng minh rằng
2mn
lcm (m, n) + lcm (m + 1, n + 1) > √
.
m−n
Lời giải. Đặt m = n + k. Khi đó ta có

(m + 1) (n + 1)
mn
+
gcd (m, n) gcd (m + 1, n + 1)
mn
mn
>

+
gcd (n + k, n) gcd (m + 1, n + 1)
mn
mn
+
=
gcd (k, n) gcd (n + k + 1, n + 1)
mn
mn
=
+
.
gcd (k, n) gcd (k, n + 1)

lcm (m, n) + lcm (m + 1, n + 1) =

Dễ thấy gcd (k, n) và gcd (k + 1, n) không thể có chung ước số nguyên
tố nào vì nếu p là một ước số nguyên tố chung của chúng thì p | n và
p | n + 1, điều này là vô lí. Mặt khác gcd (k, n) | k và gcd (k, n + 1) | k
nên ta có gcd (k, n) . gcd (k + 1, n) ≤ k.
Theo bất đẳng thức giữa trung bình cộng và trung bình nhân (AM GM) ta có
mn
mn
lcm (m, n) + lcm (m + 1, n + 1) >
+
gcd (k, n) gcd (k, n + 1)
2mn
1
1
≥ 2mn

=√
≥ 2mn
.
gcd (k, n) . gcd (k, n + 1)
k
m−n
17


Ví dụ 3.5. Cho a, b, c là các số nguyên khác không. Chứng minh rằng
(i) [a, (b, c)] = ([a, b] , [a, c]) ,
(ii) (a, [b, c]) = [(a, b) , (a, c)] .
Lời giải. (i) Ta có
(a, b, c)

c
b
,
(a, b) (a, c)

b
c
(a, b, c) ,
(a, b, c)
(a, b)
(a, c)
b
c
=
((a, b) , (b, c)) ,

((a, c) , (b, c))
(a, b)
(a, c)
b (b, c)
c (b, c)
=
b,
, c,
(a, b)
(a, c)
b (b, c)
c (b, c)
=
b,
, c,
(a, b)
(a, c)
c (b, c)
b (b, c)
,c ,
=
b,
(a, b)
(a, c)
b (b, c)
c (b, c)
=
(b, c) ,
,
(a, b)

(a, c)
b (b, c) c (b, c)
,
= (b, c) ,
(a, b) (a, c)
c
b
,
= (b, c) 1,
(a, b) (a, c)
= (b, c) .
=

Từ đó suy ra
(b, c)
=
(a, b, c)

c
b
,
.
(a, b) (a, c)

Ta có
(a, b, c) = (a, (b, c)) =


a. (b, c)
[a, (b, c)]


[a, b] c
[a, c]
b
=
;
=
(a, b)
a (a, c)
a

nên ta được
[a, (b, c)]
=
a

[a, b] [a, c]
,
a
a
18

=

1
([a, b] , [a, c]) .
a


Vậy [a, (b, c)] = ([a, b] , [a, c]) .

(ii) Từ (i) ta suy ra
a. (b, c) [a, b] . [a, c]
=
(a, b, c)
[a, b, c]
a2 bc
abc
=

(a, b, c) [b, c] (a, b) (a, c) [a, b, c]
a
1
=

(a, b, c) [b, c] (a, b) (a, c) [a, b, c]
⇔ a [b, c] (a, b, c) = (a, b) (a, c) [a, b, c]
a [b, c]
(a, b) (a, c)

=
[a, b, c]
(a, b, c)
⇔ (a, [b, c]) = [(a, b) , (a, c)]
Ví dụ 3.6. Cho ba số nguyên dương m, n và p. Chứng minh rằng
(i) [m, n, p] =

mnp (m, n, p)
.
(m, n) (n, p) (p, m)


(ii) [m, n, p] =

(m, n, p) [m, n] [n, p] [p, m]
.
mnp

Lời giải.
(i) Cách 1. Suy ra từ ví dụ 3.5 và định lí 3.1.
Cách 2. Đặt d = (m, n, p). Khi đó m = dx, n = dy, p = dz với (x, y, z) =
1.
Ta có
[m, n, p] = [dx, dy, dz] = d [x, y, z] ,
mnp (m, n, p)
dx.dy.dz.d
dxyz
=
=
.
(m, n) (n, p) (p, m) d (x, y) .d (y, z) .d (z, x) (x, y) . (y, z) . (z, x)
Do vậy. điều phải chứng minh tương đương với
xyz
[x, y, z] =
(x, y) . (y, z) . (z, x)
trong đó (x, y, z) = 1.
xy
.z
[x, y] .z
xyz
(x, y)
(3.1)

[x, y, z] = [[x, y] , z] =
=
=
.
([x, y] , z) ([x, y] , z) (x, y) . ([x, y] , z)
19


Từ đó đẳng thức (3.1) tương đương với
([x, y] , z) = (x, z) . (y, z) ,

(3.2)

với điều kiện (x, y, z) = 1.
Đặt q = (x, y). Ta có x = qx1 , y = qy1, trong đó (x1, y1) = 1. Hơn nữa
do (x, y, z) = 1 nên ta có (q, z) = 1. Do vậy đẳng thức (3.2) tương đương
(x1y1 , z) = (x1, z) . (y1 , z) .

(3.3)

Thật vậy, giả sử k = (x1y1 , z). Khi đó k | (x1y1 ) và k | z mà (x1, y1 ) = 1
nên k | x1 hoặc k | y1 . Suy ra k | (x1, z) hoặc k | (y1, z). Do đó k |
(x1, z) . (y1 , z).
Ngược lại nếu l = (x1 , z) thì l | x1 và l | z, suy ra l | x1y1 nên l | (x1y1 , z).
Tương tự (x1, z) | (x1y1 , z). Do đó (3.3) được chứng minh. Từ đó ta có
(i) được chứng minh hoàn toàn.
(ii) Suy ra từ (i) bằng cách sử dụng định lí 3.1.
Ví dụ 3.7 (USAMO 1972). Cho các số nguyên a, b và c. Chứng minh
rằng
[a, b, c]2

(a, b, c)2
=
.
[a, b] [b, c] [c, a] (a, b) (b, c) (c, a)
Lời giải.
Cách 1. Áp dụng Ví dụ 3.6 ta có
[a, b, c] =

(a, b, c) [a, b] [b, c] [c, a]
abc (a, b, c)
=
.
(a, b) (b, c) (c, a)
abc

Do đó
(a, b, c) [a, b] [b, c] [c, a] (a, b, c)2 [a, b] [b, c] [c, a]
abc (a, b, c)
.
=
.
[a, b, c] =
(a, b) (b, c) (c, a)
abc
(a, b) (b, c) (c, a)
2

Từ đó ta có điều phải chứng minh.
Cách 2. Giả sử a = pα1 1 · · · pαnn , b = pβ1 1 · · · pβnn , c = pγ11 · · · pγnn , ở đó
p1, p2, ..., pn là các số nguyên tố phân biệt, và α1 , ..., αn, β1, ..., βn, γ1, ..., γn


20


là các số nguyên không âm. Khi đó ta có
n

2 max{αi ,βi ,γi }

pi

2

[a, b, c]
=
[a, b] [b, c] [c, a]
=

i=1
n
n
max{αi ,βi }
max{βi ,γi }
max{γi ,αi }
pi
pi
pi
i=1
i=1
i=1

n
2 max{αi ,βi ,γi }−max{αi ,βi }−max{βi ,γi }−max{γi ,αi }
pi
i=1
n


n

(a, b, c)
=
(a, b) (b, c) (c, a)
=

2 min{αi ,βi ,γi }

pi

2

i=1
n
n
min{αi ,βi }
min{βi ,γi }
min{γi ,αi }
pi
pi
pi
i=1

i=1
i=1
n
2 min{αi ,βi ,γi }−min{αi ,βi }−min{βi ,γi }−min{γi ,αi }
pi
i=1
n

Ta cần chứng minh với các số nguyên không âm α, β, γ bất kì
2 max {α, β, γ} − max {α, β} − max {β, γ} − max {γ, α}

= 2 min {α, β, γ} − min {α, β} − min {β, γ} − min {γ, α} .

(3.4)

Thật vậy, do tính đối xứng, không mất tổng quát ta giả sử α ≤ β ≤ γ.
Khi đó ta có (3.4) tương đương với
2γ − β − γ − γ = 2α − α − β − α.

(3.5)

Đẳng thức (3.5) hiển nhiên đúng. Từ đó ta có điều phải chứng minh.
Chú ý: Ta thấy phương pháp này có thể dùng để chứng minh được các
ví dụ 3.5, ví dụ 3.6 theo cách tương tự.
Nhận xét 3.8. Từ ví dụ trên ta có
là các số nguyên bằng nhau.

[a, b] [b, c] [c, a]
(a, b) (b, c) (c, a)


[a, b, c]2
(a, b, c)2

Ví dụ 3.9. Cho n ≥ 2 số nguyên dương a1 , a2 , ..., an. Chứng minh rằng
a1 a2 ...an ≥ [a1 , a2, ..., an] (a1 , a2 , ..., an) .
21


Lời giải. Đặt
A=

a1 a2 ...an
.
(a1 , a2, ..., an)

Với mỗi i = 1, n ta có
A = ai .
Từ đó suy ra

a1 ...ai−1.ai+1...an ..
.ai
(a1 , a2, ..., an)

.
A.. [a1 , a2 , ..., an] .

Do đó
A ≥ [a1 , a2, ..., an] .

4


Bài tập đề nghị

Bài toán 4.1. Tìm các số tự nhiên a, b thỏa mãn (a, b) = 12, [a, b] = 532.
Bài toán 4.2. Tìm hai số a, b nguyên dương thỏa mãn a2 + b2 = 85113,
[a, b] = 1764.
Bài toán 4.3. Chứng minh rằng (a, b)n = (an , bn) với mọi số tự nhiên
n.
Bài toán 4.4. Cho các số nguyên a, b, c khác không. Chứng minh rằng
([a, b] , [b, c] , [c, a]) = [(a, b) , (b, c) , (c, a)] .
Bài toán 4.5. Chứng minh rằng
gcd
= gcd

n−1
k−1

n−1
k

,

n
k+1

,

n+1
k


,

n+1
k+1

,

n
k−1

Bài toán 4.6 (USAMO 1993). Cho a, b là các số nguyên dương lẻ. Dãy
số (fn) được xác định như sau f1 = a, f2 = b, với n ≥ 3 thì fn là ước số
lẻ lớn nhất của fn−1 + fn−2. Chứng minh rằng fn không đổi với n đủ lớn
và xác định giá trị đó theo a và b.
22


Bài toán 4.7 (1997 Canadian Mathematical Olympiad). Có bao nhiêu
cặp số nguyên dương (x, y) với x ≤ y thỏa mãn gcd (x, y) = 5! và
lcm (x, y) = 50!.
Bài toán 4.8 (1995 Russian Mathematical Olympiad). Dãy các số tự
nhiên a1 , a2 , ... thỏa mãn điều kiện gcd (ai , aj ) = gcd (i, j) với mọi i = j.
Chứng minh rằng ai = i với mọi i.
Bài toán 4.9 (AIME 1985). Cho dãy số (an ) được xác định bởi an =
100 + n2, n = 1, 2, ... Với mỗi n, đặt dn = (an , an+1). Tìm max dn .
n≥1

Tài liệu tham khảo
[1 ] Hà Huy Khoái, Chuyên đề bồi dưỡng học sinh giỏi toán trung học
phổ thông Số học, NXB Giáo dục, 2006.

[2 ] Đặng Hùng Thắng - Nguyễn Văn Ngọc - Vũ Kim Thủy, Bài giảng
số học, NXB Giáo dục Việt Nam, 2010.
[3 ] Dương Quốc Việt (chủ biên) - Đàm Văn Nhỉ, Cơ sở lí thuyết số
và đa thức, NXB Đại học Sư phạm, 2012.
[4 ] Dương Quốc Việt (chủ biên), Nguyễn Đạt Đăng, Lê Văn Đính,
Lê Thị Hà, Đặng Đình Hanh, Đào Ngọc Minh, Trương Thị Hồng
Thanh, Phan Thị Thủy, Bài tập cơ sở lí thuyết số và đa thức, NXB
Đại học Đại học Sư phạm, 2014.
[5 ] Titu Andreescu, Dorin Andrica, Zuming Feng, 104 number theory
problems, Birkhauser Boston - Basel - Berlin, 2006.
[6 ] David A. Santos, Elementary Number Theory Notes C, 2004.
[7 ] Titu Andreescu, Dorin Andrica, Number Theory Structures, Example, and Problems, Birkhauser.

23



×